Line 1: Line 1:
 +
=[[HW3_MA453Fall2008walther|HW3]], Chapter 3, Problem 9, [[MA453]], Fall 2008, [[user:walther|Prof. Walther]]=
 +
==Problem Statement==
 +
Show that U(20) does not equal <k> for any k in U(20). [Hence U(20) is not cyclic.]
 +
 +
----
 +
==Discussion==
 +
 
The problem says show that U(20) does not equal <k> for any k in U(20). [Hence U(20) is not cyclic.]  
 
The problem says show that U(20) does not equal <k> for any k in U(20). [Hence U(20) is not cyclic.]  
 
I was trying to understand Example 1 from the Chapter 3 in the text book. where it discusses U(15).
 
I was trying to understand Example 1 from the Chapter 3 in the text book. where it discusses U(15).
Line 43: Line 50:
 
----
 
----
 
Isn't this just the example from class?
 
Isn't this just the example from class?
 +
----
 +
[[HW3_MA453Fall2008walther|Back to HW3]]
 +
 +
[[Main_Page_MA453Fall2008walther|Back to MA453 Fall 2008 Prof. Walther]]

Latest revision as of 17:04, 22 October 2010

HW3, Chapter 3, Problem 9, MA453, Fall 2008, Prof. Walther

Problem Statement

Show that U(20) does not equal <k> for any k in U(20). [Hence U(20) is not cyclic.]


Discussion

The problem says show that U(20) does not equal <k> for any k in U(20). [Hence U(20) is not cyclic.] I was trying to understand Example 1 from the Chapter 3 in the text book. where it discusses U(15). I am completely confused about what it is talking about:

U(15) = {1, 2, 4, 7, 8, 11, 13, 14 } Then it goes on to say to find the order of element 7, so |7| = 4 $ 7^1 = 7space 7^2 =4 space 7^3 = 13 space 7^4 = 1 $


Let U(a) = X where X is a group with several elements. Let Z = b + n*a, so all elements in X will satisfy gcd(a,b) = 1.

So for U(15) we'll get {1, 2, 4, 7, 8, 11, 13, 14}

To find an order of an element, y in X, we just have to find a power of the modulo where it will repeat itself. So

$ 7^0 = 1 $, the remainder when 15|1 = 1

$ 7^1 = 7 $, remainder = 7

$ 7^2 = 49 $, remainder of 15|49 = 4

$ 7^3 = 343 $, remainder of 15|343 = 13

$ 7^4 = 2401 $, remainder of 15|2401 = 1

The remainder of $ 7^4 $ is the same as $ 7^0 $, and so we can end here since it the remainders will repeat itself So, the order of |7| = 4 since it has to go through 4 numbers before it repeats itself again.

To make the calculation easier, we can see that $ 7^2 = 49 = (15*n + 4) $ $ 7^3 = (15*n + 4) * 7 $ We know that $ (15*n)*7 $ is going to be divisible by 15, so we just have to find out what the remainder of $ 15|4*7 $.


I just think it's funny that the back of the book tells us to "brute force" to solve this problem. I don't think I've ever used brute force in mathematics before.  ;)


I don't get why U(20) does not equal <k> for any k in U(20). Doesn't U(20) = <1> <3> <7> <9> <11> <13> <17> <19>


check the orders


Isn't this just the example from class?


Back to HW3

Back to MA453 Fall 2008 Prof. Walther

Alumni Liaison

Ph.D. on Applied Mathematics in Aug 2007. Involved on applications of image super-resolution to electron microscopy

Francisco Blanco-Silva